Petite amusette de vacances — Les-mathematiques.net The most powerful custom community solution in the world

Petite amusette de vacances

Modifié (July 2022) dans Arithmétique
Existe-t-il deux nombres rationnels strictement positifs dont la somme est l'inverse du produit ?

Réponses

  • Modifié (July 2022)
    Ce que tu  demandes est : existe-t-il $x>0,y>0,x,y\in \mathbb{Q}$ tels que $x+y=\dfrac{1}{xy}$
    On a donc que $(x+y)^2=x^2+y^2+2xy=x^2+y^2+\dfrac{2}{x+y}=x^2+y^2+\dfrac{2(x+y)}{(x+y)^2}$ et on a amorcé un développement en fraction continue (irrégulière) infini mais dont tous les coefficients sont strictement positifs et rationnels. M'est avis que cela n'est pas possible si $x,y$ sont des rationnels.
    PS. On a par exemple :
    $\displaystyle (x+y)^2=x^2+y^2+\frac{2(x+y)}{x^2+y^2+\frac{2(x+y)}{(x+y)^2}}$.
  • On peut voir ça comme cette équation d'inconnue $y$ : $xy^2+x^2y-1=0$
    Le discriminant est $x^4+4x$, qui est positif.
    Cette équation a 2 racines réelles, et il se trouve que l'une d'elle est positive.
    Et le problème devient : quels sont les rationnels $x$ strictement positifs pour lesquels $\sqrt{x^4+4x}$ est également rationnel.
    Avantage : on n'a plus qu'une inconnue
    Inconvénient : on n'est pas plus avancé. 
    Tu me dis, j'oublie. Tu m'enseignes, je me souviens. Tu m'impliques, j'apprends. Benjamin Franklin
  • Quelqu'un va nous trouver ça avec une courbe elliptique...
  • Modifié (July 2022)
    Soit $x=\frac{a}{b}$, $y=\frac{c}{d}$, on suppose $a \wedge b=c\wedge d=1$. On montre facilement que $a \wedge c=1$.

    Soit $p$ un nombre premier.
    Soit $u=v_p(b)$, $v=v_p(c)$.
    Supposons $u=0$ et $v>0$, alors $v_p(d)=0$, donc $v_p(c/d)=v_p(c)>0$, et $v_p((bd)/(ac))\leq-v_p(c)<0$, et $v_p(a/b)\geq 0$. Donc, $v_p(a/b+c/d)\geq 0$, contradiction car $a/b+c/d=(bd)/(ac)$.
    Si $u>0$ et $v=0$, alors $v_p(a)=0$ et $v_p(a/b)=-v_p(b)<0$, et $v_p(c/d)\leq 0$, et $v_p((bd)/(ac)) \geq v_p(b)>0$. Donc $v_p(a/b)=v_p(c/d)$, donc $-v_p(b)=-v_p(d)$, donc $v_p(d)=u>0$.
    Donc $u=v=0$, ou ($u>0$ et $v>0$).

    Si $u>0$ et $v>0$, alors $v_p(a)=v_p(d)=0$. Donc $v_p(a/b)=-v_p(b)<0$, et $v_p(c/d)=v_p(c)>0$ et $v_p((bd)/(ac))= v_p(b)-v_p(c)$. Comme $a/b+c/d=(bd)/(ac)$, on a soit 1) $v_p(b)-v_p(c)=-v_p(b)$, soit 2) $v_p(c)=v_p(b)-v_p(c)$.
    Si 2), alors $a/b=(bd)/(ac) -c/d$, donc $v_p(a/b) \geq v_p(c)=v_p(b)-v_p(c) >0$, car $v>0$. Contradiction, car $v_p(a/b)<0$.
    Donc 1).
    Donc $c=b^2$.

    De même, $a=d^2$. Donc $d^2/b+b^2/d=1/(bd)$ donc $b^3+d^3=1$.
    Donc $b=0$ ou $d=0$.

    [Il y a une erreur]
  • Modifié (July 2022)
    Bonjour,
    Supposons l'existence de rationnels $x,y$ répondant à la question. Alors:
    $\exists p,q,r \in \N^* $ tels que: $\:x=\dfrac pr, \: y= \dfrac qr,\:\: p \wedge q \wedge r =1,\quad pq(p+q) = r^3.$
    Cette dernière relation fait que $p,q, p+q$ sont deux à deux premiers entre eux et donc qu'il existe $a,b,c$ dans $ \N^*$ tels que: $p =a^3 , \: q= b^3, \: p+q= c^3.$
    Cela contredit le cas le l'exposant $3$ du "grand théorème de Fermat".(qu'il est bien commode d'admettre ici car il n'a rien d'immédiat et de tout-à-fait élémentaire.)
    Ainsi, résoudre cette "amusette" revient à démontrer que $a^3+b^3 = c^3$ est impossible avec $a,b,c \in \N^*.$
  • Chouette ! Ça paraît tellement simple lorsqu'on lit ta solution ! 
  • Bravo, je soupçonnais que l'exercice ne résisterait pas très longtemps.  :)
  • Et avec trois variables $x>0,y>0,z>0$ rationnels tels que $x+y+z=\frac{1}{xyz}$ 
  • $x=\frac{1}{6}$, $y=\frac{4}{3}$, $z=\frac{3}{2}$.
  • Modifié (July 2022)
    En utilisant le raisonnement de LOU16, on peut voir que si les entiers strictement positifs $a$, $b$, $c$ et $d$ vérifient $a^4+b^4+c^4=d^4$ (il y a une infinité de solutions avec $a$, $b$, $c$ et $d$ premiers dans leur ensemble[*] : voir cet article), alors $(x\,;y\,;z)=\left(\dfrac{a^3}{bcd}\,;\dfrac{b^3}{acd}\,;\dfrac{c^3}{abd}\right)$ est solution de l'équation d'etanche.
    Mais j'ai sans doute utilisé un bulldozer pour écraser un moustique !
    ----
    [*] La "plus petite" solution est $(a\,;b\,;c\,;d)=(95800\,;217519\,;414560\,;422481)$", elle conduit à  la solution
    $(x\,;y\,;z)=\left(\dfrac{5495111950000}{238106797259649}\,;\dfrac{10291806066809359}{16778768697888000}\,;\dfrac{356231394734080}{44018971782081}\right)$ de l'équation d'etanche.

  • Modifié (July 2022)
    Pensez-vous qu'il existe des solutions $\left(x\,;y\,;z\right)$ de l'équation d'étanche différentes de celle de JLT mais moins "monstrueuses" que les précédentes ?
    Edit : oui, et il y en a une flopée ! Par exemple $\left(\dfrac{1}{6}\,;\dfrac{1}{3}\,;4\right)$ , $\left(\dfrac{7}{10}\,;\dfrac{7}{6}\,;\dfrac{18}{35}\right)$  , $\left(\dfrac{1}{6}\,;\dfrac{2}{5}\,;\dfrac{18}{5}\right)$ , $\left(\dfrac{9}{28}\,;\dfrac{8}{9}\,;\dfrac{49}{36}\right)$ , $\left(\dfrac{1}{6}\,;\dfrac{9}{28}\,;\dfrac{49}{12}\right)$ , $\left(\dfrac{7}{36}\,;\dfrac{8}{7}\,;\dfrac{14}{9}\right)$ , $\left(\dfrac{8}{45}\,;\dfrac{2}{5}\,;\dfrac{125}{36}\right)$ , $\left(\dfrac{6}{55}\,;\dfrac{22}{15}\,;\dfrac{11}{6}\right)$ , $\left(\dfrac{7}{33}\,;\dfrac{21}{22}\,;\dfrac{12}{7}\right)$ , $\left(\dfrac{9}{26}\,;\dfrac{13}{14}\,;\dfrac{26}{21}\right)$ , $\left(\dfrac{11}{21}\,;\dfrac{49}{66}\,;\dfrac{12}{11}\right)$ , $\left(\dfrac{28}{45}\,;\dfrac{7}{10}\,;\dfrac{125}{126}\right)$ , $\left(\dfrac{1}{84}\,;\dfrac{9}{28}\,;16\right)$ , $\left(\dfrac{4}{105}\,;\dfrac{28}{15}\,;\dfrac{35}{12}\right)$ , $\left(\dfrac{4}{35}\,;\dfrac{7}{30}\,;\dfrac{125}{21}\right)$ , $\left(\dfrac{2}{7}\,;\dfrac{25}{84}\,;\dfrac{63}{20}\right)$ , $\left(\dfrac{32}{105}\,;\dfrac{25}{24}\,;\dfrac{49}{40}\right)$ et $\left(\dfrac{16}{51}\,;\dfrac{75}{68}\,;\dfrac{17}{15}\right)$ .
  • Modifié (July 2022)
    Il semble qu'il y ait aussi beaucoup de solutions où $x$ est l'inverse d'un entier. Outre les précédentes, il y a :
    $\left(\dfrac{1}{204}\,;\dfrac{24}{17}\,;\dfrac{34}{3}\right)$ , $\left(\dfrac{1}{336}\,;3\,;\dfrac{147}{16}\right)$ , $\left(\dfrac{1}{546}\,;\dfrac{36}{7}\,;\dfrac{169}{21}\right)$ , $\left(\dfrac{1}{630}\,;\dfrac{20}{3}\,;\dfrac{243}{35}\right)$ ...


  • Modifié (July 2022)
    Avec n-variables $x_1>0,…,x_n>0$ rationnels $x_1+x_2+….+x_n=\frac{1}{x_1x_2…x_n}$ 

    Trois méthodes pour résoudre les équations $Q^n$ voir page 13 à 15 l’article de E.Peyre dans la Gazette des mathématiciens n159 janvier 2019 qui expliquent ces trois méthodes avec des exemples 
    https://docplayer.fr/146698544-La-gazette-des-mathematiciens-janvier-2019-n-o-159-mathematiques-les-points-rationnels-interactions-aux-racines-de-la-cognition-mathematique.html
  • Pour tout $t\in\mathbb{Q}$,

    $(x\,;y\,;z)=\left(\dfrac{6 t^3 (t^4-2)^2} {(4 t^4 + 1)   (2 t^8 + 10 t^4 - 1)}\,; \dfrac{ 3 (4 t^4 + 1)^2} {2t (t^4-2) (2 t^8 + 10 t^4 - 1)}\,;\dfrac{ 2 (2 t^8 + 10 t^4 - 1)} {3t (4 t^4 + 1)}\right)$

    est solution de l'équation $x+y+z=\dfrac{1}{xyz}\,.$

    Voir cette discussion sur Mathematics Stack Exchange.
  • Modifié (August 2022)
    Notons $(E_n)$ l'équation à solutions $(x_1\,;\cdots\,;x_n)$ dans $\mathbb{Q}^n$ :
    $$\displaystyle\sum_{k=1}^{n}x_k=\dfrac{1}{\displaystyle\prod_{k=1}^{n}x_k}\,.$$ Grâce à l'égalité $27^5+84^5+110^5+133^5=144^5$ (voir cette page), on peut voir que$$\left(\dfrac{19683}{6554240}\,;\dfrac{2744}{3135}\,;\dfrac{9150625}{2714796}\,;\dfrac{44700103}{5132160}\right)$$
    est solution de $(E_4)$.
  • Modifié (August 2022)
    Sinon, une investigation informatique permet d'obtenir les solutions suivantes de $(E_4)$ :
    $\left(\dfrac{1}{6}\,;\dfrac{1}{3}\,;2\,;2\right)$ , $\left(\dfrac{1}{6}\,;\dfrac{2}{3}\,;\dfrac{2}{3}\,;3\right)$ , $\left(\dfrac{1}{12}\,;\dfrac{1}{4}\,;\dfrac{2}{3}\,;8\right)$ , $\left(\dfrac{1}{12}\,;\dfrac{1}{4}\,;\dfrac{8}{3}\,;3\right)$ , $\left(\dfrac{1}{12}\,;\dfrac{1}{3}\,;1\,;\dfrac{16}{3}\right)$ , $\left(\dfrac{1}{10}\,;\dfrac{2}{5}\,;2\,;\dfrac{5}{2}\right)$ , $\left(\dfrac{1}{14}\,;1\,;\dfrac{7}{4}\,;\dfrac{7}{4}\right)$ , $\left(\dfrac{1}{30}\,;\dfrac{5}{6}\,;\dfrac{9}{5}\,;\dfrac{10}{3}\right)$ , $\left(\dfrac{1}{24}\,;\dfrac{9}{8}\,;\dfrac{3}{2}\,;\dfrac{8}{3}\right)$ , $\left(\dfrac{1}{4}\,;\dfrac{1}{4}\,;\dfrac{4}{3}\,;\dfrac{8}{3}\right)$ , $\left(\dfrac{1}{10}\,;\dfrac{2}{3}\,;\dfrac{9}{10}\,;\dfrac{10}{3}\right)$ , $\left(\dfrac{1}{10}\,;\dfrac{5}{6}\,;\dfrac{16}{15}\,;\dfrac{5}{2}\right)$ , $\left(\dfrac{1}{10}\,;\dfrac{16}{15}\,;\dfrac{3}{2}\,;\dfrac{3}{2}\right)$ , $\left(\dfrac{1}{10}\,;\dfrac{6}{5}\,;\dfrac{6}{5}\,;\dfrac{5}{3}\right)$ , $\left(\dfrac{1}{6}\,;\dfrac{3}{10}\,;\dfrac{6}{5}\,;\dfrac{10}{3}\right)$ , $\left(\dfrac{1}{6}\,;\dfrac{3}{5}\,;1\,;\dfrac{12}{5}\right)$ , $\left(\dfrac{2}{9}\,;\dfrac{1}{4}\,;\dfrac{16}{9}\,;\dfrac{9}{4}\right)$ , $\left(\dfrac{3}{10}\,;\dfrac{5}{6}\,;1\,;\dfrac{6}{5}\right)$ , $\left(\dfrac{2}{5}\,;\dfrac{3}{5}\,;\dfrac{5}{6}\,;\dfrac{3}{2}\right)$ , $\left(\dfrac{1}{3}\,;\dfrac{5}{12}\,;\dfrac{5}{4}\,;\dfrac{8}{5}\right)$ , $\left(\dfrac{5}{12}\,;\dfrac{1}{2}\,;\dfrac{16}{15}\,;\dfrac{27}{20}\right)$ et $\left(\dfrac{9}{20}\,;\dfrac{2}{3}\,;\dfrac{5}{6}\,;\dfrac{5}{4}\right)$.
  • Modifié (August 2022)
    Soit $a\in\Q$. En fait, j'ai réussi à trouver une paramétrisation d'un ensemble infini de solutions rationnelles de l'équation
    $$(\mathcal{E}_a) \ :\quad wxyz(w+x+y+z)=a$$
    L'idée est d'essayer de trouver une égalité polynomiale de la forme
    $$X+\lambda P^2Q^2+\mu PR^2+\varepsilon Q^2R^2=\theta P^2QR \qquad(1)$$
    avec $\lambda$, $\mu$, $\varepsilon$ et $\theta$ dans $\Q$ et $P$, $Q$ et $R$ dans $\Q[X]$.
    Posons $A=X$, $B=\lambda P^2Q^2$, $C=\mu PR^2$ et $D=\varepsilon Q^2R^2$, alors
    $$ABCD(A+B+C+D)=\lambda\mu\varepsilon\theta X(PQR)^5 \;.$$
    En prenant $X=kat^5$ avec $k\in\Q$ tel que $k\lambda\mu\varepsilon\theta$ s'écrive $K^5$ avec $K\in\Q$, nous avons 
    $$wxyz(w+x+y+z)=a \;,$$
    avec $w=\dfrac{A}{KtPQR}$, $x=\dfrac{B}{KtPQR}$, $w=\dfrac{C}{KtPQR}$ et $w=\dfrac{D}{KtPQR}$.
    En fait, on va voir qu'on peut trouver une égalité de la forme $(1)$ avec $P$, $Q$ et $R$ de degré 1.
    Prenons $P=X-\alpha$, $Q=X-\beta$ et $R=X-\gamma$ avec $\alpha$, $\beta$ et $\gamma$ dans $\Q$ (tels que $\gamma\neq\beta$).
    Supposons que $(1)$ soit vérifiée ; en remplaçant $X$ par $\alpha$, $\beta$ puis $\gamma$, en dérivant puis en remplaçant $X$ par $\alpha$, et en considérant les termes dominants, on aboutit à
    $$\left\{\begin{array}{l}\alpha+\varepsilon(\alpha-\beta)^2(\alpha-\gamma)^2=0\\\beta+\mu(\beta-\alpha)(\beta-\gamma)^2=0\\\gamma+\lambda(\gamma-\alpha)^2(\gamma-\beta)^2=0\\1+\mu(\alpha-\gamma)^2+2\varepsilon(\alpha-\beta)(\alpha-\gamma)(2\alpha-\gamma-\beta)=0\\\theta=\lambda+\varepsilon\\\end{array}\right.$$ ce qui équivaut à
    $$\left\{\begin{array}{l}\varepsilon=-\dfrac{\alpha}{(\alpha-\beta)^2(\alpha-\gamma)^2}\\\mu=-\dfrac{\beta}{(\beta-\alpha)(\beta-\gamma)^2}\\\lambda=-\dfrac{\gamma}{(\gamma-\alpha)^2(\gamma-\beta)^2}\\(\alpha-\beta)(\alpha-\gamma)(\beta-\gamma)^2+\beta(\alpha-\gamma)^3-2\alpha(\beta-\gamma)^2(2\alpha-\gamma-\beta) \;\;\;(2)\\\theta=\lambda+\varepsilon\\\end{array}\right.$$
    Posons $u=\dfrac{\alpha-\gamma}{\gamma-\beta}$ ; l'égalité $(2)$ donne
    $$(\alpha-\beta)u(\gamma-\beta)^3+\beta u^3(\gamma-\beta)^3-2\alpha(\gamma-\beta)^2(2(\alpha-\gamma)+\gamma-\beta) \;,$$
    ce qui équivaut à $$(\alpha-\beta)u+\beta u^3-2\alpha(2u+1)=0 \;,$$
    ce qui entraîne
    $$\left\{\begin{array}{l}\beta=\dfrac{3u+2}{u^3-u}\alpha\\\gamma=\dfrac{u^2+3u+1}{(1+u)(u^2-1)}\alpha\\\varepsilon=-\dfrac{u^2(u+1)^6(u-1)^4}{\alpha^3(u^3-4u-2)^4}\\\mu=\dfrac{u^2(u+1)^4(u-1)^2(3u+2)}{\alpha^2(u^3-4u-2)^3}\\\lambda=-\dfrac{u^2(u+1)^6(u-1)^3(u^2+3u+1)}{\alpha^3(u^3-4u-2)^4}\\\theta=-\dfrac{u^3(u+1)^6(u-1)^3(u+4)}{\alpha^3(u^3-4u-2)^4}\\\end{array}\right.$$
    Réciproquement, on peut vérifier que si $\beta$, $\gamma$, $\varepsilon$, $\mu$, $\lambda$ et $\theta$ sont définis en fonction de $u$ et $\alpha$ par les relations précédentes, alors la relation polynomiale $(1)$ est satisfaite.
    En prenant $k=-\dfrac{\alpha u}{(u+1)^2(u-1)^2(u^2+3u+1)(u+4)(3u+2)}$, on a $k\lambda\mu\varepsilon\theta=K^5$ avec $K=\dfrac{u^2(u+1)^4(u-1)^2}{\alpha^2(u^4-4u-2)^3}$ ; on prend alors $X=kat^5$. En posant
    $$\left\{\begin{array}{l}F(u)=(u+1)^2(u-1)^2(u^2+3u+1)(u+4)(3u+2)\\G(u)=(u+1)(u-1)(u^2+3u+1)(u+4)(3u+2)^2\\H(u)=(u-1)(u^2+3u+1)^2(u+4)(3u+2)\\J(u)=(u+4)(3u+2)(u^3-4u-2)\\\end{array}\right.$$
    on obtient (après simplifications) la $2-$paramétrisation d'un ensemble infini de solutions de $(\mathcal{E}_a)$ suivante :
    $$\left\{\begin{array}{l}w=\dfrac{a(u-1)^2(u^2+3u+1)^2(u^3-4u-2)J(u)^2t^4}{(aut^5+F(u))(au^2t^5+G(u))(aut^5+H(u))}\\x=\dfrac{(aut^5+F(u))(au^2t^5+G(u))}{u(u-1)J(u)t(aut^5+H(u))}\\y=\dfrac{u(3u+2)(aut^5+H(u))}{t(au^2t^5+G(u))}\\z=\dfrac{(au^2t^5+G(u))(aut^5+H(u))}{u(u^2+3u+1)J(u)t(aut^5+F(u))}\\\end{array}\right.$$
    En choisissant par exemple $a=1$ et  $u=3$, on obtient la paramétrisation suivante d'un ensemble infini de solutions de $(\mathcal{E}_1)$
    $$\left\{\begin{array}{l}w=\dfrac{18809562772t^4}{(3t^5+55594)(3t^5+93632)(9t^5+128744)}\\x=\dfrac{(3t^5+93632)(9t^5+128744)}{6006t(3t^5+55594)}\\y=\dfrac{33(3t^5+55594)}{t(9t^5+128744)}\\z=\dfrac{(3t^5+55594)(9t^5+128744)}{57057t(3t^5+93632)}\\\end{array}\right.$$
Connectez-vous ou Inscrivez-vous pour répondre.
Success message!